Interesting GMATFix Problem-46

This topic has expert replies
User avatar
Community Manager
Posts: 1048
Joined: Mon Aug 17, 2009 3:26 am
Location: India
Thanked: 51 times
Followed by:27 members
GMAT Score:670

Interesting GMATFix Problem-46

by arora007 » Mon Oct 04, 2010 4:21 am
x and y are positive odd integer. What is the remainder when the product of xy is divided by 18?
1)When x is divided by 9, the remainder is 3
2) y-1 is a multiple of 6
https://www.skiponemeal.org/
https://twitter.com/skiponemeal
Few things are impossible to diligence & skill.Great works are performed not by strength,but by perseverance

pm me if you find junk/spam/abusive language, Lets keep our community clean!!

User avatar
Legendary Member
Posts: 866
Joined: Mon Aug 02, 2010 6:46 pm
Location: Gwalior, India
Thanked: 31 times

by goyalsau » Mon Oct 04, 2010 4:32 am
I think it should be E,
Am i correct?
Saurabh Goyal
[email protected]
-------------------------


EveryBody Wants to Win But Nobody wants to prepare for Win.

User avatar
Legendary Member
Posts: 1460
Joined: Tue Dec 29, 2009 1:28 am
Thanked: 135 times
Followed by:7 members

by selango » Mon Oct 04, 2010 5:16 am
Clearly stmt1 and stmt2 alone not sufficient.

combining 1 and 2,

x=9a+3 (21,39,57,75...)

y-1=6b

y=6b+1 (7,13,19,25)

watever the combination we try the remainder is 3.

Pick C
--Anand--

Junior | Next Rank: 30 Posts
Posts: 27
Joined: Fri Aug 13, 2010 8:13 am
GMAT Score:650

by okletsdothis » Mon Oct 04, 2010 9:52 am
how did u get 3 ? and what is the answer here...pls always put the OA with your question.

User avatar
Master | Next Rank: 500 Posts
Posts: 233
Joined: Mon Jun 09, 2008 1:30 am
Thanked: 5 times

by blaster » Tue Oct 05, 2010 1:30 am
i don't think that answer should be 3 (it must be whether 2 or 4)

but my pick is C

Master | Next Rank: 500 Posts
Posts: 265
Joined: Mon Dec 28, 2009 9:45 pm
Thanked: 26 times
Followed by:2 members
GMAT Score:760

by mj78ind » Tue Oct 05, 2010 4:54 am
selango wrote:Clearly stmt1 and stmt2 alone not sufficient.

combining 1 and 2,

x=9a+3 (21,39,57,75...)

y-1=6b

y=6b+1 (7,13,19,25)

watever the combination we try the remainder is 3.

Pick C
It is ironic that your approach gives me an idea which proves the answer you suggest is correct but the reasoning presented by you is slightly off (I had solved it using brute force and come to C as well!)

x*y = (9a+3)*(6b+1) = 54a + 18b+9a + 3

When we divide this by 18, we get 0 remainders from the first two terms. As we know a is odd, let a = 2p+1 then the last two terms become 9*(2p+1) + 3 when divided by 18 this will always leave a remainder 12

Hence C
Call me 1 - (412) 897 6727 (US) or leave a msg on BTG for GMAT advise / questions.
If you like the solution, check out my debrief at and leave a comment:
https://www.beatthegmat.com/760-done-dea ... 66740.html

User avatar
Legendary Member
Posts: 1261
Joined: Sun Sep 14, 2008 3:46 am
Thanked: 27 times
GMAT Score:570

by reply2spg » Tue Oct 05, 2010 6:05 am
C is correct.

x = {21, 39, 57}
y = {7, 13, 19}

you will always get remainder 1 whenever you divide xy by 18.
Sudhanshu
(have lot of things to learn from all of you)

User avatar
Community Manager
Posts: 1048
Joined: Mon Aug 17, 2009 3:26 am
Location: India
Thanked: 51 times
Followed by:27 members
GMAT Score:670

by arora007 » Tue Oct 05, 2010 6:30 am
OA is C
https://www.skiponemeal.org/
https://twitter.com/skiponemeal
Few things are impossible to diligence & skill.Great works are performed not by strength,but by perseverance

pm me if you find junk/spam/abusive language, Lets keep our community clean!!

Senior | Next Rank: 100 Posts
Posts: 98
Joined: Sat Feb 06, 2010 11:56 pm
Thanked: 5 times

by anantbhatia » Tue Oct 05, 2010 8:30 am
As we know a is odd, let a = 2p+1
@MJ

why have you assumed a as odd?

User avatar
Legendary Member
Posts: 1460
Joined: Tue Dec 29, 2009 1:28 am
Thanked: 135 times
Followed by:7 members

by selango » Tue Oct 05, 2010 8:45 am
anantbhatia wrote:
As we know a is odd, let a = 2p+1
@MJ

why have you assumed a as odd?
x=9a+3

Given X is odd.So a must be even in order for X to be odd.

So a=2p

Last 2 terms=18p+3[p=1,2...]

Sub any values of p u ll get remainder as 3
--Anand--

Senior | Next Rank: 100 Posts
Posts: 98
Joined: Sat Feb 06, 2010 11:56 pm
Thanked: 5 times

by anantbhatia » Tue Oct 05, 2010 9:02 am
:(
missed out reading tht x, y are odd.

Senior | Next Rank: 100 Posts
Posts: 77
Joined: Fri Jan 08, 2010 7:55 am
Thanked: 2 times
GMAT Score:700

by pharmxanthan » Tue Oct 05, 2010 9:55 am
mj78ind wrote:
selango wrote:Clearly stmt1 and stmt2 alone not sufficient.

combining 1 and 2,

x=9a+3 (21,39,57,75...)

y-1=6b

y=6b+1 (7,13,19,25)

watever the combination we try the remainder is 3.

Pick C
It is ironic that your approach gives me an idea which proves the answer you suggest is correct but the reasoning presented by you is slightly off (I had solved it using brute force and come to C as well!)

x*y = (9a+3)*(6b+1) = 54a + 18b+9a + 3

When we divide this by 18, we get 0 remainders from the first two terms. As we know a is odd, let a = 2p+1 then the last two terms become 9*(2p+1) + 3 when divided by 18 this will always leave a remainder 12

Hence C
if we pick numbers, we ger remainder 1 when xy is divided by 18.?

Junior | Next Rank: 30 Posts
Posts: 10
Joined: Wed Aug 04, 2010 6:55 am

by [email protected] » Sat Oct 16, 2010 1:52 am
Hi..
Why cant the values of X be 12,21,30,39 and so on...

So considering this..
X=12,21,30,39....
Y=7,13,19,25 and so on...
Case 1:
xy ie. when x=12 and y =7
SO XY =84 and wen divided by 18 the remninder is 12

Case 2: x=21 and y =7
xy= 147 and wen divided by 18 the reminder is 3.
Pls help.
hz do i answer this.

Junior | Next Rank: 30 Posts
Posts: 10
Joined: Wed Aug 04, 2010 6:55 am

by [email protected] » Sat Oct 16, 2010 1:54 am
oops sorry sorry.. mistake.. i got it.. oits oly odd positive integers.. ahh..